Đến nội dung

Hình ảnh

Tôpic nhận đề Hình học


  • Chủ đề bị khóa Chủ đề bị khóa
Chủ đề này có 11 trả lời

#1
E. Galois

E. Galois

    Chú lùn thứ 8

  • Quản lý Toán Phổ thông
  • 3861 Bài viết

Topic này dùng để BTC nhận đề thi từ các toán thủ thi đấu về Hình học

 

I- Bạn cần biết:

1) Điều lệ giải đấu

2) Lịch thi đấu

3) Đăng kí thi đấu

 

II - Yêu cầu về đề bài
1. Hình thức:

- Đề bài phải có đáp án kèm theo.

- Đề bài và đáp án được gõ $\LaTeX$ rõ ràng

2. Nôi dung

* Đối với MSS

- Mỗi bộ đề bao gồm 1 câu của THCS. Kiến thức dùng để giải bài không vượt quá kiến thức thi tuyển sinh vào 10. 

- Đề bài không được ở dạng thách đố, cách giải ngặt ngèo thông qua những bổ đề quá khó, không copy nguyên văn từ đề thi tuyển sinh vào 10, Olympic hoặc HSG cấp tỉnh trở lên.

- Toán thủ không nên copy đề bài từ một topic nào đó của VMF, không được post lại đề đã nộp ra topic mới dù cho đề có được chọn hay không.

 

III - Mẫu đăng kí và nộp đề

1. Họ và tên thật:

2. Đang học lớp ?, trường ?, huyện ?, tỉnh ?

3. Đề 

4. Đáp án

 

IV - Chú ý

1) Bạn sẽ thấy ở trên khung trả lời của bạn có dòng sau Bài viết này phải qua kiểm duyệt của quản trị viên mới được đăng lên diễn đàn.

Điều này có nghĩa là các toán thủ khi nộp đề, cứ yên tâm rằng, sau khi đánh máy và ấn nút GỬI BÀI là đề đã được lưu, BTC đã nhận được đề của bạn, có điều bạn không nhìn thấy được mà thôi. Bạn nên mừng vì điều này, như thế các toán thủ khác không thể biết trước đề của bạn được.

 

2) Bạn cũng nên sử dụng chức năng xem trước của diễn đàn để sửa các lỗi $\LaTeX$ trước khi gửi bài, vì gửi rồi sẽ không xem và sửa lại được nữa. 

 

3) Nếu đề bài của bạn không được chấp nhận, BTC sẽ làm hiện nó và nói rõ lý do vì sao, khi đó, bạn phải nộp đề khác. 

Nếu đề bài của bạn được chấp nhận, bạn sẽ thấy tên mình trong danh sách thi đấu tại đây sau mỗi thứ 7 hàng tuần.

 

4) Mỗi tuần, BTC chỉ cho phép toán thủ đăng kí 1 nộp đề cho 1 chủ đề nên bạn đừng ngạc nhiên khi thấy có lúc topic này bị khóa

 

 


1) Xem cách đăng bài tại đây
2) Học gõ công thức toán tại: http://diendantoanho...oạn-thảo-latex/
3) Xin đừng đặt tiêu đề gây nhiễu: "Một bài hay", "... đây", "giúp tớ với", "cần gấp", ...
4) Ghé thăm tôi tại 
http://Chúlùnthứ8.vn

5) Xin đừng hỏi bài hay nhờ tôi giải toán. Tôi cực gà.


#2
duongld

duongld

    Binh nhì

  • Thành viên
  • 10 Bài viết

Họ và tên:Nguyễn Mạnh Trùng Dương

Đang học lớp 8/1 THCS Long Định Châu Thành Tiền Giang 

Đề:Cho tam giác $ABC$ vuông tại $C$ đường cao $CH$. Đường tròn $(I;r)$ nội tiếp $ABC$ tiếp xúc với $AB,AC$ lần lượt tại $P,Q$. Giao điểm của $CH$ và $PQ$ là $N$. Gọi $K$ là trung điểm của $BC, KI$ cắt $AC$ tại $M$
chứng minh:$CM=CN$
Đáp án: Qua $C$ kẻ đường thẳng vuông góc $NH$ cắt $PQ$ tại $E$

Ta cần chứng minh tam giác $CEN$ và tam giác $PIA$ bằng nhau$
Ta có $CE$ song song $AB$ (cùng vuông $NH$
nên tam giác $CEP$ và tam giác $AQP$ đồng dạng 
do đó tam giác $CEP$ cân tại $C$
Suy ra $CE=CP=IP(1)$
Và góc $CEP=$ góc $AQP=$ góc $AIP(2)$
Từ (1) và (2) ta có tam giác $CEN$ và tam giác $PIA$ bằng nhau 
Do đó $CN=PA$ mặt khác ta chứng minh được rằng $AM=PC=r$
Suy ra $AP=CM$
suy ra $CM=CN$


Nguyễn Mạnh Trùng Dương tự hào là thành viên của VMF

Mời các mem Sài Gòn tham gia quán trà đá của anh Badman tại đây

#3
Super Fields

Super Fields

    Thiếu úy

  • Thành viên
  • 526 Bài viết

1/ Họ và tên thật: Lê Vũ Minh Huy

 

2/ Đang học lớp 8A, trường THCS Trần Quốc Toản, thành phố Tuy Hòa, tỉnh Phú Yên

 

3/ Đề:

 

Cho đoạn thẳng AB. M;N $\in AB$ (M nằm giữa A,N). Vẽ các $\Delta$ đều :$\Delta AMD$ ; $\Delta MNE$ ;$\Delta NBF$. G là trọng tâm $\Delta DEF$. GH $\perp$

 AB ( H $\in$ AB). C/minh: độ dài GH không phụ thuộc vào vị trí điểm M và N trên AB

 

4/ Đáp án:

 

Để giải bài toán này, cần chứng minh thêm một bài toán phụ:

 

$\Delta ABC$ có G là trọng tâm. Đường thẳng d cắt các cạnh của $\Delta ABC$ . $AA'\perp d ; GG' \perp d ; CC'\perp d ; BB'\perp d$.Chứng minh :$GG'=\frac{AA' + BB' +CC'}{3}$

 

Hình vẽ của bài toán phụ:

 File gửi kèm  7.png   103.85K   0 Số lần tải

 Lấy E là trung điểm BG ; Hạ $EE' \perp B'C'$

Tứ giác ACC'A'' là hình thang vuông ( $\widehat{A'} = \widehat{C'} = 90^{\circ}$)

Tứ giác BGG'B' là hình thang vuông ( $\widehat{B'} = \widehat{G'} = 90^{\circ}$)

Tứ giác EFF'E' là hình thang vuông ( $\widehat{E'} = \widehat{F'} = 90^{\circ}$)

Trong hình thang BGG'B' có $BE = EG = \frac{1}{2}BG$ ; EE' // BB' // GG' => $B'E' = E'G' = \frac{1}{2}B'G'$

Vì vậy ta có : EE' là đường trung bình của hình thang BGG'B'

C/m tương tự ở hình thang EFF'E' => GG' là đường trung bình của hình thang EFF'E'

                      và hình thang ACC'A' =>FF' là  đường trung bình của hình thang ACC'A'

 Sử dụng tính chất đường trung bình trong hình thang:

=>2GG' = EE' + FF'

=>4GG' =2EE' +2FF'

=>4GG' = BB' + GG' +AA' + CC'

=> 3GG' =AA' +BB' +CC'

=> ĐPCM

+Trở về bài toán chính:

File gửi kèm  6.png   121.82K   0 Số lần tải

Hạ $DD' \perp AB ; EE' \perp AB ;FF' \perp AB$

Theo kết quả bài toán phụ, ta có:

$GH=\frac{DD'+EE'+FF'}{3}$

DD' là đường cao của $\Delta ADM$ đều => $DD' = \frac{AM\sqrt{3}}{2}$

Tương tự => $EE' = \frac{MN\sqrt{3}}{2}$

                      $FF' = \frac{NB\sqrt{3}}{2}$

=>$GG' = \frac{AB\sqrt{3}}{6}$

 


$\dagger$God made the integers, and else is the work of man.$\dagger$


$\boxed{\textrm{My Blog}}$


#4
nghiemthanhbach

nghiemthanhbach

    $\sqrt{MF}'s\;friend$

  • Thành viên
  • 1056 Bài viết

Tên: Nghiêm Thanh Bách

Lớp 9a4, trường THCS Trường Chinh Quận Tân Bình TPHCM

Đề:

Cho$\Delta ABC$ vuông tại $A$ có $BC=a$, đường cao AH. $(O,r);(O_{1},r_{1});(O_{2},r_{2})$ là đường tròn nội tiếp của $\Delta ABC,\Delta ABH,\Delta ACH$.

a) Chứng minh: $r_{1}+r_{2}+r=AH$

b) $r_{2}^2+r_{1}^2= r^2$

c) $S_{ABC}$$\leq \frac{(a+2r)^2}{8}$

 

Đap án:

a) Gọi O là tâm đường tròn nội tiếp tam giác $ABC$ cắt $ BC,AC,AB$ lần lượt tại D,E,F và $(O_{1})$ cắt AB tại T

Ta có:$OD = r$ và $O_{1}T=r_{1}$ $\rightarrow \widehat{EAO}=\frac{1}{2}\widehat{EAF}=45^o$

nên tam giác AEO vuông cân E $\rightarrow AF=AE=OE=r$

ta có điều này tự chứng minh rất dễ (anh cho phép em làm tắt ạ, phần này dài lắm anh ơi :()

$r=\frac{AB+AC-BC}{2}$;$r_{1}=\frac{AH-AB+BH}{2}$;$r_{2}=\frac{AH+CH-AC}{2}$

Cộng lại với nhau ra điều phải chứng minh

b) Dễ dàng chứng minh được điều sau đây: $\widehat{BAO_{1}}=\frac{1}{2}\widehat{BAH}=\frac{1}{2}\widehat{ACB}=\widehat{OCB}\rightarrow$ $B;O_{1};O$ thẳng hàng.

Chứng minh $\Delta O_{1}BA\sim \Delta OBC\rightarrow \frac{O_{1}T}{OD}=\frac{AB}{BC}=\frac{r_{1}}{r}$ (Anh thông cảm vì em không tìm được cái đồng dạng)

Chứng minh tương tự $\rightarrow \frac{r_{2}}{r}=\frac{AC}{BC}$

$\rightarrow \frac{r_{1}^2+r_{2}^2}{r}=\frac{AB^2+AC^2}{BC}=1$

Vậy là ra điều phải chứng minh.

c) Ta có: $\frac{AB+AC-BC}{2}=r\rightarrow AB+AC=2r+a\rightarrow S_{ABC}=\frac{1}{2}ABAC\leq \frac{1}{2}\frac{(AB+AC)^2}{2}=\frac{(2r+a)^2}{8}$

Áp dụng bất đẳng thức cauchy



#5
Taisaokhong

Taisaokhong

    Binh nhì

  • Thành viên
  • 11 Bài viết

1. Họ và tên thật: Phan thị Thu Hiền

2. Đang học lớp 9C , trường THCS Nguyễn Cao, huyện  Quế Võ, tỉnh Bắc Ninh

3. Đề: Cho tam giác ABC có 3 góc nhọn,nội tiếp đương tròn tâm O. Các đường cao AD và CE của tam giác ABC cắt nhau tại H.  CM: a) tứ giác EHDK nội tiếp 

4. Đáp án Có        

                      Góc HEB = 90 độ ( CE la đường cao )

                      Góc HDB= 90 độ( AD là đường cao)

 

Suy ra góc HEB cộng góc HDB = 180 độ

Suy ra Tứ giác EHDB nội tiếp ( E, H đối nhau)

 

                     

 

 

 

 

 

 

 



#6
Viet Hoang 99

Viet Hoang 99

    $\textbf{Trương Việt Hoàng}$

  • Điều hành viên THPT
  • 2291 Bài viết
Đề:
Một đường thẳng đi qua tâm của đường tròn nội tiếp tam giác $ABC$ cắt $AB;AC$ lần lượt tại $M;N$.
Chứng minh: $\frac{BM.CN}{AM.AN}\leq \frac{BC^{2}}{4AB.AC}$
Giải:
Gọi bán kính đường tròn nội tiếp là $r$
$S_{AIM}+S_{AIN}=\frac{S_{AMN}}{S_{ABC}}.S_{ABC}$
$\Leftrightarrow \frac{1}{2}r.AM+\frac{1}{2}r.AN=\frac{AM.AN}{AB.AC}.\frac{1}{2}r(AB+BC+CA)$
$\Leftrightarrow AM+AN=\frac{AM.AN}{AB.AC}$
$\Leftrightarrow \frac{AM.AB.AC}{AM.AN}+\frac{AN.AB.AC}{AM.AN}=AB+BC+CA$
$\Leftrightarrow \frac{AB.AC}{AN}-AB+\frac{AB.AC}{AM}-AC=BC$
$\Leftrightarrow \frac{AB(AC-AN)}{AN}+\frac{AC(AB-AM)}{AM}=BC$
$\Leftrightarrow \frac{AB.NC}{AN}+\frac{AC.BM}{AM}=BC$
$\Leftrightarrow \frac{AB.NC}{AN.BC}+\frac{AC.BM}{AM.BC}=1$
Áp dụng bất đẳng thức Cauchy cho 2 số dương ta có:
$1=\frac{AB.NC}{AN.BC}+\frac{AC.BM}{AM.BC}\geq 2\sqrt{\frac{AB.NC}{AN.BC}.\frac{AC.BM}{AM.BC}}$
$\Leftrightarrow \frac{4AB.AC.BM.CN}{AM.AN.BC^{2}}\leq 1$
$\Leftrightarrow \frac{BM.CN}{AM.AN}\leq \frac{BC^{2}}{4AB.AC}$ (đpcm)
Dấu bằng có khi: $MN$ là đường trung bình tam giác $ABC$.


Đề:
Tam giác $ABC$ nhọn nội tiếp $(O;R)$. Ba cạnh $BC;CA;AB$ có độ dài là $a;b;c$. $I$ nằm trong tam giác $ABC$. $x;y;z$ là khoảng cách từ $I$ đến $a;b;c$.
Chứng minh: $\sqrt{x}+\sqrt{y}+\sqrt{z}\leq \sqrt{\frac{a^{2}+b^{2}+c^{2}}{2R}}$
Giải:
Có: $S=\frac{abc}{4R}$
$\Leftrightarrow 2R=\frac{abc}{2S}$
$\Leftrightarrow \frac{a^{2}+b^{2}+c^{2}}{2R}=\frac{(a^{2}+b^{2}+c^{2}).2S}{abc}$
$\rightarrow \frac{a^{2}+b^{2}+c^{2}}{2R}\geq \frac{(ab+bc+ca)(ax+by+cz)}{abc}=(\sum \frac{1}{a})(ax+by+cz)\geq (\sum \sqrt{x})^{2}(B.C.S)$
Ta có đpcm

#7
congtudung999

congtudung999

    Lính mới

  • Thành viên
  • 7 Bài viết

Tên:Phan Công Dũng

Lớp 9B Trường THCS Bình Thịnh Huyện Đức Thọ-Hà Tĩnh

Đề:Cho tam giác ABC. Một đường tròn (O) qua A và B cắt AC và BC ở D và E. M là giao điểm thứ hai của đường tròn ngoại tiếp các tam giác ABC và DEC. Chứng minh rằng góc OMC vuông.

Chỉ dẫn:

Đặt $ \widehat{CAB}=\widehat{CED}=\alpha$

Nếu $ \widehat{CAB}<\widehat{ABC}$ thì M nằm trong$ \widehat{BAC}$.Ta có $ \widehat{CMD}= \alpha$,$ \widehat{CMB}= 180^{\circ}-\alpha$,$ \widehat{DMB}= 180^{\circ}-2\alpha$.$ \widehat{DOB}= 2\alpha$$ \Rightarrow \widehat{DMB}+\widehat{DOB}=180^{\circ}$$ \Rightarrow$ tứ giác DMBO nội tiếp. Vì DO=B,MO là phân giác của$ \widehat{DMB}$. Như vậy :

$ \widehat{CMO}=\widehat{CMD}+\frac{1}{2}\widehat{DMB}=\alpha +\frac{1}{2}(180^{\circ}-2\alpha )=90^{\circ}$

 



#8
dennydo99

dennydo99

    Lính mới

  • Thành viên
  • 3 Bài viết

1.Họ và tên thật: Đỗ Ngọc Sơn

2.Đang học lớp :9A, trường: Thcs Cao Viên, huyện Thanh Oai, thành phố Hà Nội

3.Đề:Cho đường tròn (O), dây AB và dây CD, AB<CD. Giao điểm K của các đường thẳng AB,CD nằm ngoài đường tròn. Đường tròn (O;OK) cắt KA và KC tại M và N.

CM: KM<KN

4.Đáp án: kẻ OI vuông góc AB, OE vuông góc CD

trong đường tròn nhỏ:

AB<CD suy ra OI>OE.

Trong đường tròn lớn:

OI>OE suy ra KM<KN


:icon10: ok  :icon10:


#9
lethanhson2703

lethanhson2703

    Thượng sĩ

  • Thành viên
  • 297 Bài viết

Họ và tên: Nguyễn Thị Mỹ Linh

Học sinh lớp 8a

Trường THCS Lãng CÔng, Huyện Sông Lô, Tỉnh VĨnh Phúc

Cho $\widehat{xÂy}$ nhọn, B và C lần lượt là các điểm cố định trện Ax và Ay sao cho AB<AC. M là điểm di chuyển trong $\widehat{xÂy}$ sao cho $\frac{MA}{MB}=\frac{1}{2}$ . Xác định vị trí của M sao cho MB+2MC min

⇒$\Rightarrow 2MA=MB$$\Rightarrow 2MA=MB$

post-95599-0-63082700-1341501437.gif 

 

$\Rightarrow 2MA=MB$

$MB+2MC\Leftrightarrow 2(Ma+MC))\geq 2AC$

Dấu bằng xảy ra $\Leftrightarrow M\epsilon AC$ sao cho 2MA=MB



#10
phamphucat

phamphucat

    Hạ sĩ

  • Thành viên
  • 77 Bài viết

Họ và tên: Nguyễn Văn Phẩm

Lớp: 9a7 Trường: Trường Thcs Ngô Mây Quận/Huyện: Huyện Phù Cát Tỉnh/Thành phố: Bình Định

Đề: Cho tam giác ABC có $\angle A=50^0;\angle B=20^0$. Trên đường phân giác BE của $\angle ABC$ lấy điểm F sao cho $\angle FAB=20^0$. Gọi I là trung điểm của AF, K là giao điểm của tia EI với AB và M là giao điểm của CK với EB.

Chứng minh rằng: $AI^2 +EI^2=AE(MF+\frac{1}{2}EK)$

Đáp án:

AE=EF. Tam giác AEI vuông tại I;  $\angle KEB=60^0. AI^2+EI^2=AE^2=AE.EF=AE.(MF+EM)=AE.(MF+\frac{1}{2}EK)$


photo-116227.png?_r=1377943765


#11
minhlong02121999

minhlong02121999

    Binh nhất

  • Thành viên
  • 37 Bài viết

1) Họ và tên thật: Lê Nguyễn Minh Long

2) Đang học lớp 9/1, trường THCS Nguyễn Trường Tộ, thành phố Vĩnh Long, tỉnh Vĩnh Long

3) Đề: 

  a) Chứng minh rằng $\forall n\in N$ chẵn thì $A=20^{n}+16^{n}-3^{n}-1\vdots 323$

  b) Tìm nghiệm nguyên của bất phương trình $x^{2}+y^{2}+z^{2}< xy+3y+2z-3$

  c) Tìm số dư của B=$776^{776}+777^{777}+778^{778}$ khi chia cho 3

4) Đáp án:

  a) Ta có $A=20^{n}+16^{n}-3^{n}-1=(20^{n}-1)+(16^{n}-3^{n})$

      Mà $20^{n}-1\vdots (20-1)=19$ và n chẵn nên $16^{n}-3^{n}\vdots (16+3)=19$

      $\rightarrow 20^{n}+16^{n}-3^{n}-1\vdots 19$ (1)

      Mặt khác $A=20^{n}+16^{n}-3^{n}-1=(20^{n}-3^{n})+(16^{n}-1)$

      Mà $20^{n}-3^{n}\vdots (20-3)=17$ và n chẵn nên $16^{n}-1\vdots (16+1)=17$

      $\rightarrow 20^{n}+16^{n}-3^{n}-1\vdots 17$ (2)

      Từ (1),(2) và (17,19)=1 $\Rightarrow 20^{n}+16^{n}-3^{n}-1\vdots (17.19)=323$

      Vậy $A\vdots 323\forall n\in N$ (n chẵn)

  b) Ta có $x^{2}+y^{2}+z^{2}< xy+3y+2z-3$

      $\Leftrightarrow x^{2}+y^{2}+z^{2}-xy-3y-2z+3< 0$

      $\Leftrightarrow x^{2}+y^{2}+z^{2}-xy-3y-2z+3\leqslant -1$

      $\Leftrightarrow (x^{2}-xy+\frac{y^{2}}{4})+3(\frac{y^{2}}{4}-y+1)+z^{2}-2z+1\leqslant 0$

      $\Leftrightarrow (x-\frac{y}{2})^{2}+3(\frac{y}{2}-1)^{2}+(z-1)^{2}\leqslant 0$

      $\Leftrightarrow \left\{\begin{matrix}x-\frac{y}{2} =0 \\\frac{y}{2}-1=0 \\ z-1=0 \end{matrix}\right.$

      $\Leftrightarrow \left\{\begin{matrix}x=1 \\y=2 \\ z=1 \end{matrix}\right.$

      Vậy bất phương trình có nghiệm duy nhất (1,2,1)

  c) Ta có $776\equiv -1(mod 3) \Rightarrow 776^{776}\equiv (-1)^{776} (mod3)\Rightarrow 776^{776}\equiv 1(mod3)$

               $777\equiv 0(mod3)\Rightarrow 777^{777}\equiv 0(mod3)$

               $778\equiv 1(mod3)\Rightarrow 778^{778}\equiv 1(mod3)$

      $\Rightarrow 776^{776}+777^{777}+778^{778}\equiv 1+0+1=2(mod3)$

      Vậy B chia 3 dư 2



#12
KimAn98

KimAn98

    Lính mới

  • Thành viên
  • 3 Bài viết

Cho tam giác ABC cân tại A nội tiếp đường tròn tâm O bán kính R. ta kẻ dây AM cắt dây BC tại N

Chứng minh tích AM.AN=2R.h

Lời giải

 

File gửi kèm  Ảnh chụp màn hình (3).png   138.86K   0 Số lần tải

 

$\angle AMB=\angle ACB$ (Chắn cung AB)

MÀ $\angle ACB=\angle ABC$

Vậy $\angle AMB=\angle ABN$ hay$\angle AMB=\angle ABN$

$\Rightarrow$ tam giác ABN đồng dạng tam giác AMB

$\Rightarrow \frac{AB}{AM}=\frac{AN}{AB}$

$\Rightarrow AM.AN=AB^2$ (không đổi)

Kẻ đường kính AD cắt BC tại H theo hệ thức lượng trong tam giác vuông ABD ta có:

$AB^2=AD.AH$ hay $AB^2=2R.h$

Vậy tích AM.AN=2Rh

 






1 người đang xem chủ đề

0 thành viên, 1 khách, 0 thành viên ẩn danh